Poisson and convergence in distribution











up vote
1
down vote

favorite
1












Let $X_1,X_2,...$ be iid with mean $mu$ and variance $sigma^2$. Let $N_{lambda}$ be poisson($lambda$) independent of the $X_i$'s.



(a) Find the limit in distribution as $lambda rightarrow infty$ for $$frac{sum_{i=1}^{N_{lambda}}X_i-N_{lambda}mu}{sqrt{lambda}}.$$



(b) Find the limit in distribution as $lambda rightarrow infty$ for $$frac{sum_{i=1}^{N_{lambda}}X_i-lambdamu}{sqrt{lambda}}.$$



(c) For which random variables $X$ will the two limits be the same?



My solution:
Let $Y_{lambda}$ be the quotient given in the problem. Then $mathbb{E}e^{it Y_{lambda}}=sum_{n=0}^{infty}mathbb{E}(e^{itY_{lambda}}1_{N_{lambda}=n})=sum_{n=0}^{infty}(phi_{X'_1}(frac{t}{lambda} ))^nfrac{e^{-lambda} lambda^n}{n!}=sum_{n=0}^{infty}(1-sigma^2frac{t^2}{2lambda^2}+o(frac{t^2}{lambda^2}))^n frac{e^{-lambda}lambda^n}{n!}$



where $X'=X-mu$. Can the above sum be simplified as $lambda$ goes to $infty$?










share|cite|improve this question


























    up vote
    1
    down vote

    favorite
    1












    Let $X_1,X_2,...$ be iid with mean $mu$ and variance $sigma^2$. Let $N_{lambda}$ be poisson($lambda$) independent of the $X_i$'s.



    (a) Find the limit in distribution as $lambda rightarrow infty$ for $$frac{sum_{i=1}^{N_{lambda}}X_i-N_{lambda}mu}{sqrt{lambda}}.$$



    (b) Find the limit in distribution as $lambda rightarrow infty$ for $$frac{sum_{i=1}^{N_{lambda}}X_i-lambdamu}{sqrt{lambda}}.$$



    (c) For which random variables $X$ will the two limits be the same?



    My solution:
    Let $Y_{lambda}$ be the quotient given in the problem. Then $mathbb{E}e^{it Y_{lambda}}=sum_{n=0}^{infty}mathbb{E}(e^{itY_{lambda}}1_{N_{lambda}=n})=sum_{n=0}^{infty}(phi_{X'_1}(frac{t}{lambda} ))^nfrac{e^{-lambda} lambda^n}{n!}=sum_{n=0}^{infty}(1-sigma^2frac{t^2}{2lambda^2}+o(frac{t^2}{lambda^2}))^n frac{e^{-lambda}lambda^n}{n!}$



    where $X'=X-mu$. Can the above sum be simplified as $lambda$ goes to $infty$?










    share|cite|improve this question
























      up vote
      1
      down vote

      favorite
      1









      up vote
      1
      down vote

      favorite
      1






      1





      Let $X_1,X_2,...$ be iid with mean $mu$ and variance $sigma^2$. Let $N_{lambda}$ be poisson($lambda$) independent of the $X_i$'s.



      (a) Find the limit in distribution as $lambda rightarrow infty$ for $$frac{sum_{i=1}^{N_{lambda}}X_i-N_{lambda}mu}{sqrt{lambda}}.$$



      (b) Find the limit in distribution as $lambda rightarrow infty$ for $$frac{sum_{i=1}^{N_{lambda}}X_i-lambdamu}{sqrt{lambda}}.$$



      (c) For which random variables $X$ will the two limits be the same?



      My solution:
      Let $Y_{lambda}$ be the quotient given in the problem. Then $mathbb{E}e^{it Y_{lambda}}=sum_{n=0}^{infty}mathbb{E}(e^{itY_{lambda}}1_{N_{lambda}=n})=sum_{n=0}^{infty}(phi_{X'_1}(frac{t}{lambda} ))^nfrac{e^{-lambda} lambda^n}{n!}=sum_{n=0}^{infty}(1-sigma^2frac{t^2}{2lambda^2}+o(frac{t^2}{lambda^2}))^n frac{e^{-lambda}lambda^n}{n!}$



      where $X'=X-mu$. Can the above sum be simplified as $lambda$ goes to $infty$?










      share|cite|improve this question













      Let $X_1,X_2,...$ be iid with mean $mu$ and variance $sigma^2$. Let $N_{lambda}$ be poisson($lambda$) independent of the $X_i$'s.



      (a) Find the limit in distribution as $lambda rightarrow infty$ for $$frac{sum_{i=1}^{N_{lambda}}X_i-N_{lambda}mu}{sqrt{lambda}}.$$



      (b) Find the limit in distribution as $lambda rightarrow infty$ for $$frac{sum_{i=1}^{N_{lambda}}X_i-lambdamu}{sqrt{lambda}}.$$



      (c) For which random variables $X$ will the two limits be the same?



      My solution:
      Let $Y_{lambda}$ be the quotient given in the problem. Then $mathbb{E}e^{it Y_{lambda}}=sum_{n=0}^{infty}mathbb{E}(e^{itY_{lambda}}1_{N_{lambda}=n})=sum_{n=0}^{infty}(phi_{X'_1}(frac{t}{lambda} ))^nfrac{e^{-lambda} lambda^n}{n!}=sum_{n=0}^{infty}(1-sigma^2frac{t^2}{2lambda^2}+o(frac{t^2}{lambda^2}))^n frac{e^{-lambda}lambda^n}{n!}$



      where $X'=X-mu$. Can the above sum be simplified as $lambda$ goes to $infty$?







      real-analysis probability-theory characteristic-functions poisson-process






      share|cite|improve this question













      share|cite|improve this question











      share|cite|improve this question




      share|cite|improve this question










      asked Nov 19 at 6:11









      S_Alex

      788




      788






















          1 Answer
          1






          active

          oldest

          votes

















          up vote
          1
          down vote













          Note that $sum_{n=0}^infty z^nmathbb P(N_lambda =n)=e^{lambda(z-1)}$ is a probability generating function of $N_lambda$. You need also to replace $lambda$ in denominator by $sqrt{lambda}$. Then
          $$mathbb{E}e^{it Y_{lambda}}=sum_{n=0}^{infty}mathbb{E}(e^{itY_{lambda}}1_{N_{lambda}=n})=sum_{n=0}^{infty}left(phi_{X'_1}left(frac{t}{sqrt{lambda}}right)right)^nfrac{e^{-lambda} lambda^n}{n!}
          $$

          $$
          =e^{lambdaleft(phi_{X'_1}left(frac{t}{sqrt{lambda}}right)-1right)}=
          e^{lambdaleft(1-sigma^2frac{t^2}{2lambda}+oleft(frac{t^2}{lambda}right)-1right)} to e^{-frac{t^2sigma^2}{2}}.
          $$






          share|cite|improve this answer





















            Your Answer





            StackExchange.ifUsing("editor", function () {
            return StackExchange.using("mathjaxEditing", function () {
            StackExchange.MarkdownEditor.creationCallbacks.add(function (editor, postfix) {
            StackExchange.mathjaxEditing.prepareWmdForMathJax(editor, postfix, [["$", "$"], ["\\(","\\)"]]);
            });
            });
            }, "mathjax-editing");

            StackExchange.ready(function() {
            var channelOptions = {
            tags: "".split(" "),
            id: "69"
            };
            initTagRenderer("".split(" "), "".split(" "), channelOptions);

            StackExchange.using("externalEditor", function() {
            // Have to fire editor after snippets, if snippets enabled
            if (StackExchange.settings.snippets.snippetsEnabled) {
            StackExchange.using("snippets", function() {
            createEditor();
            });
            }
            else {
            createEditor();
            }
            });

            function createEditor() {
            StackExchange.prepareEditor({
            heartbeatType: 'answer',
            convertImagesToLinks: true,
            noModals: true,
            showLowRepImageUploadWarning: true,
            reputationToPostImages: 10,
            bindNavPrevention: true,
            postfix: "",
            imageUploader: {
            brandingHtml: "Powered by u003ca class="icon-imgur-white" href="https://imgur.com/"u003eu003c/au003e",
            contentPolicyHtml: "User contributions licensed under u003ca href="https://creativecommons.org/licenses/by-sa/3.0/"u003ecc by-sa 3.0 with attribution requiredu003c/au003e u003ca href="https://stackoverflow.com/legal/content-policy"u003e(content policy)u003c/au003e",
            allowUrls: true
            },
            noCode: true, onDemand: true,
            discardSelector: ".discard-answer"
            ,immediatelyShowMarkdownHelp:true
            });


            }
            });














            draft saved

            draft discarded


















            StackExchange.ready(
            function () {
            StackExchange.openid.initPostLogin('.new-post-login', 'https%3a%2f%2fmath.stackexchange.com%2fquestions%2f3004584%2fpoisson-and-convergence-in-distribution%23new-answer', 'question_page');
            }
            );

            Post as a guest















            Required, but never shown

























            1 Answer
            1






            active

            oldest

            votes








            1 Answer
            1






            active

            oldest

            votes









            active

            oldest

            votes






            active

            oldest

            votes








            up vote
            1
            down vote













            Note that $sum_{n=0}^infty z^nmathbb P(N_lambda =n)=e^{lambda(z-1)}$ is a probability generating function of $N_lambda$. You need also to replace $lambda$ in denominator by $sqrt{lambda}$. Then
            $$mathbb{E}e^{it Y_{lambda}}=sum_{n=0}^{infty}mathbb{E}(e^{itY_{lambda}}1_{N_{lambda}=n})=sum_{n=0}^{infty}left(phi_{X'_1}left(frac{t}{sqrt{lambda}}right)right)^nfrac{e^{-lambda} lambda^n}{n!}
            $$

            $$
            =e^{lambdaleft(phi_{X'_1}left(frac{t}{sqrt{lambda}}right)-1right)}=
            e^{lambdaleft(1-sigma^2frac{t^2}{2lambda}+oleft(frac{t^2}{lambda}right)-1right)} to e^{-frac{t^2sigma^2}{2}}.
            $$






            share|cite|improve this answer

























              up vote
              1
              down vote













              Note that $sum_{n=0}^infty z^nmathbb P(N_lambda =n)=e^{lambda(z-1)}$ is a probability generating function of $N_lambda$. You need also to replace $lambda$ in denominator by $sqrt{lambda}$. Then
              $$mathbb{E}e^{it Y_{lambda}}=sum_{n=0}^{infty}mathbb{E}(e^{itY_{lambda}}1_{N_{lambda}=n})=sum_{n=0}^{infty}left(phi_{X'_1}left(frac{t}{sqrt{lambda}}right)right)^nfrac{e^{-lambda} lambda^n}{n!}
              $$

              $$
              =e^{lambdaleft(phi_{X'_1}left(frac{t}{sqrt{lambda}}right)-1right)}=
              e^{lambdaleft(1-sigma^2frac{t^2}{2lambda}+oleft(frac{t^2}{lambda}right)-1right)} to e^{-frac{t^2sigma^2}{2}}.
              $$






              share|cite|improve this answer























                up vote
                1
                down vote










                up vote
                1
                down vote









                Note that $sum_{n=0}^infty z^nmathbb P(N_lambda =n)=e^{lambda(z-1)}$ is a probability generating function of $N_lambda$. You need also to replace $lambda$ in denominator by $sqrt{lambda}$. Then
                $$mathbb{E}e^{it Y_{lambda}}=sum_{n=0}^{infty}mathbb{E}(e^{itY_{lambda}}1_{N_{lambda}=n})=sum_{n=0}^{infty}left(phi_{X'_1}left(frac{t}{sqrt{lambda}}right)right)^nfrac{e^{-lambda} lambda^n}{n!}
                $$

                $$
                =e^{lambdaleft(phi_{X'_1}left(frac{t}{sqrt{lambda}}right)-1right)}=
                e^{lambdaleft(1-sigma^2frac{t^2}{2lambda}+oleft(frac{t^2}{lambda}right)-1right)} to e^{-frac{t^2sigma^2}{2}}.
                $$






                share|cite|improve this answer












                Note that $sum_{n=0}^infty z^nmathbb P(N_lambda =n)=e^{lambda(z-1)}$ is a probability generating function of $N_lambda$. You need also to replace $lambda$ in denominator by $sqrt{lambda}$. Then
                $$mathbb{E}e^{it Y_{lambda}}=sum_{n=0}^{infty}mathbb{E}(e^{itY_{lambda}}1_{N_{lambda}=n})=sum_{n=0}^{infty}left(phi_{X'_1}left(frac{t}{sqrt{lambda}}right)right)^nfrac{e^{-lambda} lambda^n}{n!}
                $$

                $$
                =e^{lambdaleft(phi_{X'_1}left(frac{t}{sqrt{lambda}}right)-1right)}=
                e^{lambdaleft(1-sigma^2frac{t^2}{2lambda}+oleft(frac{t^2}{lambda}right)-1right)} to e^{-frac{t^2sigma^2}{2}}.
                $$







                share|cite|improve this answer












                share|cite|improve this answer



                share|cite|improve this answer










                answered Nov 20 at 2:40









                NCh

                6,0882722




                6,0882722






























                    draft saved

                    draft discarded




















































                    Thanks for contributing an answer to Mathematics Stack Exchange!


                    • Please be sure to answer the question. Provide details and share your research!

                    But avoid



                    • Asking for help, clarification, or responding to other answers.

                    • Making statements based on opinion; back them up with references or personal experience.


                    Use MathJax to format equations. MathJax reference.


                    To learn more, see our tips on writing great answers.





                    Some of your past answers have not been well-received, and you're in danger of being blocked from answering.


                    Please pay close attention to the following guidance:


                    • Please be sure to answer the question. Provide details and share your research!

                    But avoid



                    • Asking for help, clarification, or responding to other answers.

                    • Making statements based on opinion; back them up with references or personal experience.


                    To learn more, see our tips on writing great answers.




                    draft saved


                    draft discarded














                    StackExchange.ready(
                    function () {
                    StackExchange.openid.initPostLogin('.new-post-login', 'https%3a%2f%2fmath.stackexchange.com%2fquestions%2f3004584%2fpoisson-and-convergence-in-distribution%23new-answer', 'question_page');
                    }
                    );

                    Post as a guest















                    Required, but never shown





















































                    Required, but never shown














                    Required, but never shown












                    Required, but never shown







                    Required, but never shown

































                    Required, but never shown














                    Required, but never shown












                    Required, but never shown







                    Required, but never shown







                    Popular posts from this blog

                    Quarter-circle Tiles

                    build a pushdown automaton that recognizes the reverse language of a given pushdown automaton?

                    Mont Emei